The answer is n 2 n^2

Algebra Level 4

31 2 α 1 + 31 2 α 2 + 31 2 α 3 + 31 2 α 4 \large \frac{31}{2- \alpha_1}+ \frac{31}{2- \alpha_2}+\frac{31}{2- \alpha_3} +\frac{31}{2- \alpha_4}

Given that 1 , α 1 , α 2 , α 3 , α 4 1,\alpha_1, \alpha_2, \alpha_3,\alpha_4 are distinct fifth roots of unity, evaluate the expression above.


The answer is 49.

This section requires Javascript.
You are seeing this because something didn't load right. We suggest you, (a) try refreshing the page, (b) enabling javascript if it is disabled on your browser and, finally, (c) loading the non-javascript version of this page . We're sorry about the hassle.

10 solutions

Pratik Shastri
Oct 31, 2014

As 1 , α 1 , , α 4 1,\alpha_1, \cdots ,\alpha_4 are roots of the equation x 5 1 = 0 , x^5-1=0, ( x 1 ) i = 1 4 ( x α i ) = x 5 1 i = 1 4 ( x α i ) = x 5 1 x 1 \begin{aligned} (x-1)\prod_{i=1}^{4} (x-\alpha_i)&=x^5-1\\ \prod_{i=1}^{4} (x-\alpha_i)&=\dfrac{x^5-1}{x-1} \end{aligned}

Take the natural logarithm on both the sides and then differentiate wrt x x .

i = 1 4 log ( x α i ) = log ( x 5 1 ) log ( x 1 ) i = 1 4 1 x α i = 5 x 4 x 5 1 1 x 1 \begin{aligned} \sum_{i=1}^{4} \log(x-\alpha_i)&=\log(x^5-1)-\log(x-1)\\ \sum_{i=1}^{4} \dfrac{1}{x-\alpha_i}&=\dfrac{5x^4}{x^5-1}-\dfrac{1}{x-1} \end{aligned} Substitute x = 2 x=2 in the above obtained equality to see that i = 1 4 1 2 α i = 49 31 \sum_{i=1}^{4} \dfrac{1}{2-\alpha_i}=\dfrac{49}{31} i = 1 4 31 2 α i = 49 \therefore \sum_{i=1}^{4} \dfrac{31}{2-\alpha_i}=\boxed{49}

Great way for those who are familiar with manipulating with calculus.

There is another more intuitive approach, which looks at finding the polynomial whose roots are 2 α i 2 - \alpha_i . Can anyone add that as a solution?

Calvin Lin Staff - 6 years, 7 months ago

If instead of 2 - a_(i), we have 1 - a(i) Then

Rajat Raj - 6 years, 5 months ago

Log in to reply

@rajat raj Take the limit.

Pratik Shastri - 6 years, 5 months ago

nice solution dude.

Trishit Chandra - 6 years, 3 months ago

WTF this is awesome! Would've never thought of it this way! :D (Even though there is a step which involves calculus, which I've completely forgotten...) MASSIVE UPVOTE!

A Former Brilliant Member - 4 years, 9 months ago

Great solution dude.. Awsome I loved it.

Nivedit Jain - 4 years, 2 months ago

So, the value of n is 7..because ''the answer is n squared''! (just joking!)

Yoogottam Khandelwal - 5 years, 11 months ago
Sandeep Bhardwaj
Oct 31, 2014

if a 1 , a 2 , . . . . . a n a_1,a_2,.....a_n are the roots of f ( x ) = 0 f(x)=0 , then f ( x ) f ( x ) = i = 1 n 1 x a i . \dfrac{f'(x)}{f(x)}=\sum_{i=1}^n \dfrac{1}{x-a_i}.

Considering f ( x ) = x 5 1 f(x)=x^5-1 here and using the above property, we have

5 x 4 x 5 1 = 1 x 1 + 1 x α 1 + 1 x α 2 + 1 x α 3 + 1 x α 4 . \dfrac{5x^4}{x^5-1}=\dfrac{1}{x-1}+\dfrac{1}{x-\alpha_1}+\dfrac{1}{x-\alpha_2}+\dfrac{1}{x-\alpha_3}+\dfrac{1}{x-\alpha_4}.

Putting x = 2 x=2 we get

80 31 = 1 1 + 1 2 α 1 + 1 2 α 2 + 1 2 α 3 + 1 2 α 4 \dfrac{80}{31}=\dfrac{1}{1}+\dfrac{1}{2-\alpha_1}+\dfrac{1}{2-\alpha_2}+\dfrac{1}{2-\alpha_3}+\dfrac{1}{2-\alpha_4}

31 2 α 1 + 31 2 α 2 + 31 2 α 3 + 31 2 α 4 = 49 . \implies \dfrac{31}{2-\alpha_1}+\dfrac{31}{2-\alpha_2}+\dfrac{31}{2-\alpha_3}+\dfrac{31}{2-\alpha_4}=\boxed{49}.

Interesting! I never knew this property of polynomials

Ariel Gershon - 6 years, 7 months ago

Can you prove that the property of the polynomials always holds Sandeep?

Omkar Kamat - 6 years, 5 months ago

Log in to reply

Since over complex numbers, a equation of degree n n always has n n roots, it can be written as f ( x ) = i = 1 n ( x a i ) f(x) = \prod_{i=1}^n (x-a_i)

Differenciating using the product rule, we find that the differenciated result is a sum of many terms where in each term, one of the ( x a i ) (x-a_i) is missing. f ( x ) = ( x a 2 ) ( x a 3 ) ( x a 4 ) ( x a n ) + ( x a 1 ) ( x a 3 ) ( x a 4 ) ( x a n ) + ( x a 1 ) ( x a 2 ) ( x a 4 ) ( x a n ) + + ( x a 1 ) ( x a 2 ) ( x a 3 ) ( x a n 1 ) = i = 1 n f ( x ) x a i f'(x) = (x-a_2)(x-a_3)(x-a_4)\dots(x-a_n) + (x-a_1)(x-a_3)(x-a_4)\dots(x-a_n) + (x-a_1)(x-a_2)(x-a_4)\dots(x-a_n) + \dots + (x-a_1)(x-a_2)(x-a_3)\dots(x-a_{n-1}) = \sum_{i=1}^n \frac{f(x)}{x-a_i} Hence the result follows. (It also holds when the leading coefficient isn't 1)

Aritra Das - 5 years, 3 months ago

@Sandeep Bhardwaj Best solution

Shubhendra Singh - 6 years, 7 months ago

Log in to reply

thank you dude for the appreciation. :)

Sandeep Bhardwaj - 6 years, 7 months ago
Ayush Verma
Oct 31, 2014

Calculus method is best & easiest but i am posting a different method.

1 + x + x 2 + x 3 + x 4 = ( x α 1 ) ( x α 2 ) ( x α 3 ) ( x α 4 ) L e t x = 2 t & 2 α = β P = 1 + ( 2 t ) + ( 2 t ) 2 + ( 2 t ) 3 + ( 2 t ) 4 & ( 2 t α 1 ) ( 2 t α 2 ) ( 2 t α 3 ) ( 2 t α 4 ) = ( t β 1 ) ( t β 2 ) ( t β 3 ) ( t β 4 ) P = ( t β 1 ) ( t β 2 ) ( t β 3 ) ( t β 4 ) 1 β 1 + 1 β 2 + 1 β 3 + 1 β 4 = β 1 β 2 β 3 β 1 β 2 β 3 β 4 = c o f f . o f t i n P c o n s t . t e r m i n P = ( 1 ) { 1 2 1 ( 2 1 ) 2 2 ( 3 1 ) 2 3 ( 4 1 ) } 1 + 2 + 2 2 + 2 3 + 2 4 = 49 31 31 2 α 1 + 31 2 α 2 + 31 2 α 3 + 31 2 α 4 = 49 1+x+{ x }^{ 2 }+{ x }^{ 3 }+{ x }^{ 4 }=\left( x-{ \alpha }_{ 1 } \right) \left( x-{ \alpha }_{ 2 } \right) \left( x-{ \alpha }_{ 3 } \right) \left( x-{ \alpha }_{ 4 } \right) \\ \\ Let\quad x=2-t\quad \& 2-\alpha =\beta \\ \\ P=1+\left( 2-t \right) +{ \left( 2-t \right) }^{ 2 }+{ \left( 2-t \right) }^{ 3 }+{ \left( 2-t \right) }^{ 4 }\\ \\ \& \left( 2-t-{ \alpha }_{ 1 } \right) \left( 2-t-{ \alpha }_{ 2 } \right) \left( 2-t-{ \alpha }_{ 3 } \right) \left( 2-t-{ \alpha }_{ 4 } \right) \\ \\ \quad \quad \quad =\left( t-{ \beta }_{ 1 } \right) \left( t-{ \beta }_{ 2 } \right) \left( t-{ \beta }_{ 3 } \right) \left( t-{ \beta }_{ 4 } \right) \\ \\ \quad \quad \therefore P=\left( t-{ \beta }_{ 1 } \right) \left( t-{ \beta }_{ 2 } \right) \left( t-{ \beta }_{ 3 } \right) \left( t-{ \beta }_{ 4 } \right) \\ \\ \therefore \cfrac { 1 }{ { \beta }_{ 1 } } +\cfrac { 1 }{ { \beta }_{ 2 } } +\cfrac { 1 }{ { \beta }_{ 3 } } +\cfrac { 1 }{ { \beta }_{ 4 } } =\cfrac { \sum { { \beta }_{ 1 } } { \beta }_{ 2 }{ \beta }_{ 3 } }{ { \beta }_{ 1 }{ \beta }_{ 2 }{ \beta }_{ 3 }{ \beta }_{ 4 } } \\ \\ \quad \quad =\cfrac { -coff.\quad of\quad t\quad in\quad P }{ const.\quad term\quad in\quad P } \\ \\ \quad =\left( -1 \right) \cfrac { \left\{ -1-{ 2 }^{ 1 }\left( \begin{matrix} 2 \\ 1 \end{matrix} \right) { -2 }^{ 2 }\left( \begin{matrix} 3 \\ 1 \end{matrix} \right) -{ 2 }^{ 3 }\left( \begin{matrix} 4 \\ 1 \end{matrix} \right) \right\} }{ 1+2+{ 2 }^{ 2 }+{ 2 }^{ 3 }+{ 2 }^{ 4 } } \\ \\ \quad =\cfrac { 49 }{ 31 } \\ \\ \therefore \cfrac { 31 }{ 2-{ \alpha }_{ 1 } } +\cfrac { 31 }{ 2-{ \alpha }_{ 2 } } +\cfrac { 31 }{ 2-{ \alpha }_{ 3 } } +\cfrac { 31 }{ 2-{ \alpha }_{ 4 } } =49

Pranshu Gaba
Dec 27, 2014

The required expression can be rewritten as i = 1 4 2 5 α i 5 2 α i \displaystyle\sum _{i = 1} ^{4} \dfrac{2^5 - \alpha_i ^5 }{2 - \alpha_i} .

We see that x 5 a 5 x a = x 4 + a x 3 + a 2 x 2 + a 3 x + a 4 \dfrac{x^5 - a^5}{x - a} = x^4 + ax^3 + a^2 x^2 + a^3 x + a^4 . After substituting x = 2 x = 2 and a = α i a = \alpha_i , the required expression becomes i = 1 4 ( 16 + 8 α i + 4 α i 2 + 2 α i 3 + α i 4 ) \displaystyle\sum _{i = 1} ^{4} (16 + 8 \alpha_i + 4 \alpha_i ^2 + 2 \alpha_i^3 + \alpha_i^4)

Since i = 1 4 α i = i = 1 4 α i 2 = i = 1 4 α i 3 = i = 1 4 α i 4 = 1 \displaystyle\sum _{i=1} ^{4} \alpha _i = \displaystyle\sum _{i=1} ^{4} \alpha _i^2 = \displaystyle\sum _{i=1} ^{4} \alpha _i^3 = \displaystyle\sum _{i=1} ^{4} \alpha _i^4 = -1 , the expression becomes 16 × 4 + ( 1 ) ( 8 + 4 + 2 + 1 ) = 64 15 = 49 16 \times 4 + (-1)(8 + 4 + 2 + 1) = 64- 15 = \boxed{49}

superb solution!! :D really ingenious !

Aritra Jana - 6 years, 5 months ago

Log in to reply

Thanks :) I noticed that no one else had used the fact that 31 = 2 5 1 31=2^5-1 , so I posted this solution.

Pranshu Gaba - 6 years, 5 months ago

Log in to reply

I used this exact same method! High Five!

Kishore S. Shenoy - 5 years, 9 months ago

Elegant solution. Relatively, I spent too much time on a brutal way. XD

Bo-Chien Huang - 5 years ago
Aritra Jana
Nov 1, 2014

I have solved this by calculus, but instead i will be posting a different solution.

Since α 1 , α 2 , α 3 , α 4 \alpha_1 ,\alpha_2 ,\alpha_3 ,\alpha_4 are four of the five 5 t h 5^{th} roots of unity, we have:

P ( x ) = x 4 + x 3 + x 2 + x + 1 \large{P(x)=x^{4}+x^{3}+x^{2}+x+1} which have the roots: α 1 , α 2 , α 3 , α 4 \alpha_1 ,\alpha_2 ,\alpha_3 ,\alpha_4

Note that we need to find 31. 1 2 x k \large{31.\sum{\frac{1}{2-x_k}}} where x k x_k are the roots of P ( x ) P(x) .

Let: y k = 1 2 x k \large{y_k=\frac{1}{2-x_k}} so, we need to find y k \sum{y_k}

A short computation yields x k = 2 y k 1 y k \large{x_k=\frac{2y_k-1}{y_k}}

Plugging this in P ( x ) P(x) , we get:

( 2 y k 1 y k ) 4 + ( 2 y k 1 y k ) 3 + ( 2 y k 1 y k ) 2 + 2 y k 1 y k + 1 = 0 \large{(\frac{2y_k-1}{y_k})^{4}+(\frac{2y_k-1}{y_k})^{3}+(\frac{2y_k-1}{y_k})^{2}+\frac{2y_k-1}{y_k}+1=0}

Multiplying by y k 4 y_k^{4} , we get

y k 4 + y k 3 ( 2 y k 1 ) + y k 2 ( 2 y k 1 ) 2 + y k ( 2 y k 1 ) 3 + ( 2 y k 1 ) 4 \large{y_k^{4}+y_k^{3}(2y_k-1)+y_k^{2}(2y_k-1)^{2}+y_k(2y_k-1)^{3}+(2y_k-1)^{4}}

It is easy to observe that :

coefficeint of \text{coefficeint of} y k 4 = 1 + 2 + 2 2 + 2 3 + 2 4 = 31 y_k^{4}=1+2+2^{2}+2^{3}+2^{4}=31

coefficeint of \text{coefficeint of} y k 3 = ( 1 + 2 . 2 C 1 + 2 2 . 3 C 2 + 2 3 . 4 C 3 ) = 49 y_k^{3}=-(1+2.^{2}C_1+2^{2}.^{3}C_2+2^{3}.^{4}C_3)=-49

y k = 49 31 = 49 31 \large{\therefore \sum{y_k}=-\frac{-49}{31}=\frac{49}{31}}

31. 1 2 x k = 31. y k = 49 \implies \large{31.\sum{\frac{1}{2-x_k}}=31.\sum{y_k}=\boxed{49}}


@Calvin Lin . i think you were thinking of something like this? :D

That's one way to get the polynomial, via a change of variables, so that the roots are y = 1 2 x y = \frac{1}{ 2 - x } .

I also like Ayush's approach above, where he made the roots β = 2 α \beta = 2 - \alpha , and then proceeded to find 1 β \sum \frac{1}{ \beta } .

Calvin Lin Staff - 6 years, 7 months ago

Log in to reply

汶良 林 - 5 years, 11 months ago

Log in to reply

i did it the same way. brilliant solution!

Prakhar Bindal - 5 years, 6 months ago

yes, it was a good solution

Aritra Jana - 6 years, 7 months ago
Kartik Sharma
Dec 25, 2014

First of all, this method is long and not advisable too but just I am sharing for clarity.

Well, we can see that 5th root of unity means α 5 = 1 {\alpha}^{5} = 1

α 5 = e 2 n ι π {\alpha}^{5} = {e}^{2n\iota\pi}

α = e 2 n ι π 5 \alpha = {e}^{\frac{2n\iota\pi}{5}}

So, α 1 = e 2 ι π 5 {\alpha}_{1} = {e}^{\frac{2\iota\pi}{5}}

α 2 = e 4 ι π 5 {\alpha}_{2} = {e}^{\frac{4\iota\pi}{5}}

α 3 = e 6 ι π 5 {\alpha}_{3} = {e}^{\frac{6\iota\pi}{5}}

α 4 = e 8 ι π 5 {\alpha}_{4} = {e}^{\frac{8\iota\pi}{5}}

α 5 = e 10 ι π 5 = e 2 ι π = 1 {\alpha}_{5} = {e}^{\frac{10\iota\pi}{5}} = {e}^{2\iota\pi}= 1

And similarly, α 6 = e 12 ι π 5 {\alpha}_{6} = {e}^{\frac{12\iota\pi}{5}} , but wait,

e 12 ι π 5 = e 2 ι π + 2 ι π 5 = e 2 ι π 5 {e}^{\frac{12\iota\pi}{5}} = {e}^{2\iota\pi + \frac{2\iota\pi}{5}} = {e}^{\frac{2\iota\pi}{5}}

So, we got our 4 5 t h 5th roots of unity. Hence, substituting,

31 ( 1 2 e 2 ι π 5 + 1 2 e 4 ι π 5 + 1 2 e 6 ι π 5 + 1 2 e 8 ι π 5 ) 31(\frac{1}{2- {e}^{\frac{2\iota\pi}{5}}} + \frac{1}{2- {e}^{\frac{4\iota\pi}{5}}} + \frac{1}{2- {e}^{\frac{6\iota\pi}{5}}} + \frac{1}{2- {e}^{\frac{8\iota\pi}{5}}})

We know that e ι x = c o s x + ι s i n x {e}^{\iota x} = cos x + \iota sin x

Now, using some trigonometry, we can further simplify it as -

31 ( 1 + 15 6 ( c o s 4 π 5 + c o s 2 π 5 ) 25 20 ( c o s 4 π 5 + c o s 2 π 5 ) + 16 c o s 2 π 5 c o s 4 π 5 31(1 + \frac{15 - 6(cos\frac{4\pi}{5} + cos\frac{2\pi}{5})}{25 - 20(cos\frac{4\pi}{5} + cos\frac{2\pi}{5}) + 16cos\frac{2\pi}{5}cos\frac{4\pi}{5}}

We know that -

c o s 4 π 5 + c o s 2 π 5 = 1 2 cos\frac{4\pi}{5} + cos\frac{2\pi}{5} = -\frac{1}{2}

c o s 2 π 5 c o s 4 π 5 = 1 4 cos\frac{2\pi}{5}cos\frac{4\pi}{5} = -\frac{1}{4}

Substituting both these in the above equation -

31 ( 1 + 18 31 ) = 49 31(1 + \frac{18}{31}) = \boxed{49}

I hope this method to be the easiest..

The question involves Complex Numbers. Then, why can't we use complex guys?

Notice one thing

Let a general solution of the roots be α \alpha

Fifth root of unity then can be

G i v e n 1 α 1 α 2 α 3 α 4 R e n a m e 1 α α 2 α 3 α 4 \displaystyle\begin{array}{l|c|r} Given&1 &\alpha_1&\alpha_2&\alpha_3&\alpha_4\\ \hline Rename&1&\alpha&\alpha^2&\alpha^3&\alpha^4\end{array}

And they're cyclic (or whatever term it is called)

Also notice

i = 1 4 α i = α = α 2 = α 3 = α 4 \displaystyle\sum_{i=1}^4 \alpha_i = \sum \alpha = \sum \alpha^2= \sum \alpha^3= \sum \alpha^4\\

Now begins the FIGHT

31 = 2 5 α 5 31 2 α = 2 5 α 5 2 α = 2 4 + 2 3 α + 2 2 α 2 + 2 α 3 + α 4 \\\displaystyle 31 = 2^5 - \alpha^5\\ \Rightarrow\begin{aligned}\frac{31}{2-\alpha} &=\frac{2^5-\alpha^5}{2 - \alpha}\\&=2^4 + 2^3 \alpha+2^2\alpha^2+2 \alpha^3+\alpha^4\\\end{aligned}\\

1 4 2 5 α 5 2 α = 4 2 4 + 2 3 ( α ) + 2 2 ( α 2 ) \nonumber + 2 ( α 3 ) + ( α 4 ) \displaystyle\begin{aligned}\Rightarrow \sum \limits_1^4 \frac{2^5 - \alpha^5}{2-\alpha} &= 4\cdot 2^4 + 2^3\left(\sum \alpha\right) + 2^2\left(\sum \alpha^2\right) \nonumber \\ &\qquad{} +2\left(\sum \alpha^3\right)+\left(\sum \alpha^4 \right)\end{aligned}\\

To get α \displaystyle \sum\alpha , take the equation

x 5 = 1 x^5=1 OR x 5 1 = 0 x^5-1=0

1 + α = 0 α = 1 α = α 2 = α 3 = α 4 = 1 \begin{aligned}\Rightarrow 1+\sum\alpha &= 0\\\Rightarrow \sum\alpha &= -1 \\\Rightarrow\sum \alpha &= \sum \alpha^2= \sum \alpha^3= \sum \alpha^4=-1\end{aligned}

Hence the answer,

1 4 2 5 α 5 2 α = 4 2 4 2 3 2 2 2 1 = 64 8 4 2 1 = 49 \displaystyle\begin{aligned}\Rightarrow \sum \limits_1^4 \frac{2^5 - \alpha^5}{2-\alpha} &= 4\cdot2^4 - 2^3 - 2^2-2-1\\&=64-8-4-2-1\\&=49\end{aligned}

Answer is 49 \huge\therefore \text{ Answer is }\boxed{49}

Moderator note:

It is typically hard to really demonstrate that a particular method is the easiest.

For example, my approach would be to create a polynomial whose roots are 1 2 α i \frac{1}{2 - \alpha_i } , and then apply Veita's to find the sum of roots. That, to me, is much easier, especially once you understand how to extend this concept to other such similar cases.

In other cases, if the number of number of roots of unity given is odd, you can use this method.

Kishore S. Shenoy - 5 years, 9 months ago

How is sigma alpha = sigma (alpha)^2 and so on. I couldn't understand that part.

Puneet Pinku - 5 years ago
Ravi Dwivedi
Jul 8, 2015

Thanks for the nice question.

Moderator note:

The approach that you took is somewhat long-winded, and could be simplified greatly. The idea is indeed to find a polynomial whose roots are ( 2 α i ) ( 2 - \alpha_i ) .

Note: Can you clarify the last line? I'm not sure how you obtained 49 from it.

We need to calculate sum of roots of g(x) taken three at a time because when I have added 1,2,3,4 see what I got as E. By vieta's relation this is equal to -(coeff ofx)/coeff of x^3 It may be long winded but I will say this approach is unique and different from what others have posted here so that we get a new solution.

Ravi Dwivedi - 5 years, 11 months ago

Log in to reply

Basically scaling of roots is used.

Ravi Dwivedi - 5 years, 11 months ago
Patrick Bamba
May 31, 2016

Let n = 31 2 a n=\dfrac{31}{2-a}

Some algebra yields a n = 2 n 31 an=2n-31

Raising both sides of this equation by 5, we get

n 5 = 32 n 5 5 2 4 31 n 4 + . . . n^5=32n^5-5*2^431n^4+...

By Vieta's , the sum of the five values of n n is 5 2 4 31 31 = 80 \dfrac{5*2^4*31}{31}=80

But why are there five values of n when there are only four terms in our problem? This is because we included the non-existent term n = 31 2 1 = 31 n=\dfrac{31}{2-1}=31 when we raised the equation to the 5th power.

Thus, the sum of the four terms in the problem is

80 31 = 49 80-31=\boxed{49}

Nice but why is the sum of values divided by 31 and not 32 when sum of the values is -b/a??

Puneet Pinku - 5 years ago

Log in to reply

because we have an n 5 n^5 term at the LHS

Patrick Bamba - 4 years, 10 months ago
汶良 林
Jul 17, 2015

Write a solution.

0 pending reports

×

Problem Loading...

Note Loading...

Set Loading...